sum physics doubts

Q1. A point moves in a plane so that its angular velocity about two fixed points is the same in the plane. The point describes:
(a) circle (b) straight line (c) ellipse (d) parabola

Q2. St 1: If the law of gravitation becomes inverse cube law, even then a line joining from the sun to the planet sweeps out equal area in equal intervals of time.

St. 2: A planet moves in an elliptical path around the sun.

19 Answers

106
Asish Mahapatra ·

Q3. A body moving along horizontal direction with momentum p, strikes the free end of a spring kept on a smooth horizontal surface. The other end of the spring is fixed to a rigid support. The mean momentum of the body after striking till it produces maximum compression in the spring will be?

Yeh mean momentum kya hai?

1
ANKIT MAHATO ·

i think both are correct ... just chill ..

106
Asish Mahapatra ·

but i think it will be valid for st. line too

1
Kalyan Pilla ·

Why cant it be b. straight line??

The perpendicular distance between the path of point of locus and the two stationary points is equal and a constant

So the angular velocity of the point will remain equal about both the points.

ω= v/r

So Y is that not supposed to be the answer

1
ANKIT MAHATO ·

ya but how can mean momentum be dimensionless ...

1
ANKIT MAHATO ·

yaar yeh i kahan se aa gaya ..

106
Asish Mahapatra ·

its given (Î /4)*p

1
ANKIT MAHATO ·

momentum .. initial is p
final is 0 .. then max compression ...
so mean i think is p/2 ...

1
ANKIT MAHATO ·

as u could see Ip is initial position of point and Fp is final position of point ... C , D are the two points around which angular velocity is to be fount ...
If it takkes time t to go from Ip to Fp .. let this extend an angle θ at C ... according to property of circle ... at D also θ is subtended .. therefore rhe angular velocity is same about C and D = θ/t ... got it yaar ... [50]

1
ANKIT MAHATO ·

sorry .. power failure .. i am giving answer for the first ...

1
Optimus Prime ·

Q.1 answer is (c)ellipse

q.2 asnwer is b i.e both statements are correct but S-II is not correct explanation of statement I

106
Asish Mahapatra ·

bas itni se main kuch samajh nahiin sakta

1
ANKIT MAHATO ·

for the first one it is a cirlcle and the two points are on the circumference of the circle ...

106
Asish Mahapatra ·

@ankit... 2. ka tumne pehle d phir c phir d kahaa mujhe toh a lag rahaa hai.

yes Q1. is circle.. but plz explain

1
ANKIT MAHATO ·

If the law of gravitation becomes inverse cube law .. it will no longer follows keplers 1st and second law ... which is valid only for n<3 ..
here both assertion and reason is false ...

1
ANKIT MAHATO ·

1 -> circle ...

106
Asish Mahapatra ·

explain (yes it is d)

1
ANKIT MAHATO ·

answer of 2 is d ..

106
Asish Mahapatra ·

no amit both answers are wrong :(

Your Answer

Close [X]